Compound Binomial-Exponential: Closed form for the PDF?












4












$begingroup$


Setup:



Consider the random variable $Y_N$ derived from
$$Y_N = sum_{i=0}^N X_i$$
where $N$ is a random variable with distribution $p_n = {M choose n} p^n q^{M-n}$ (binomial i.e. $M$ Bernoulli trials) and the $X_i$ are random variables with distribution $f(x) = lambda e^{-lambda x}$ (exponential).
I'd like to get the pdf of $y$.
Previously I asked this question which led me to expect the distribution is the sum
$$p(y) = sum_{i=1}^M {M choose i} p^i q^{M-i} frac{lambda^iy^{i-1}e^{-lambda y}}{(i-1)!} $$
or the integral (inverse Fourier transform of the characteristic function)
$$p(y) = int_{-infty}^{infty} frac{dk}{2pi} e^{-iky}(frac{plambda}{lambda-ik}+q)^M = int_{-infty}^{infty} frac{dk}{2pi}e^{-iky}(frac{ilambda + kq}{ilambda + k})^M$$
The second equality follows from $p+q=1$.
I'd like to solve either one of these problems in closed form if possible. The moment generating function these derive from is
$$ p(s) = (frac{plambda}{lambda-s}+q)^M.$$
The characteristic function is $p(s=ik)$.



Question:



What is the distribution $p(y)$ of $Y_N$?



Attempts:



I tried binomial expanding $p(s)$ in the fourier transform form, which leads to a sum over integrals like $int_{-infty}^{infty} du u^{-m}e^{-a u}$ where $m$ is a positive integer -- almost a sum of gamma functions but with negative parameters and over the wrong limits.



I tried manipulaing the sum form into something I could use the binomial formula on, but I can't obtain binomial coefficients from the factorial prefactors in the sum.



Any thoughts? I had thought a compound binomial-exponential process would have been well studied with a closed form solution but I couldn't find anything immediately relevant.



Other:



This paper explores a very similar problem but I'll have to read carefully to see how it relates (link).










share|cite|improve this question











$endgroup$








  • 1




    $begingroup$
    Your definition of the random number is a little confusing is it N or (as it appears in usage) n or i?
    $endgroup$
    – herb steinberg
    Dec 14 '18 at 22:24










  • $begingroup$
    $M$ is a parameter of the binomial distribution (it was $N$ before), i.e. the number of Bernoulli trials of probabilities $p$ and $q$ which contribute to the distribution. $N$ is a random variable, and $n$ is a particular realization of that random variable. Edited thanks @MikeEarnest
    $endgroup$
    – kevinkayaks
    Dec 14 '18 at 22:31










  • $begingroup$
    The question has no answer without any independence assumptions.
    $endgroup$
    – Kavi Rama Murthy
    Dec 14 '18 at 23:28










  • $begingroup$
    The $X_i$ are all independent and identically distributed. $X_i$ and $N$ are independent. These assumptions are embedded into the two expressions I gave for $p(y)$.
    $endgroup$
    – kevinkayaks
    Dec 14 '18 at 23:35








  • 1




    $begingroup$
    Your first expression for $p(y)$ can be written $e^{-lambda y}frac{q^M}ysum_{i=0}^M binom{M}i frac{t^i}{(i-1)!}$, where $t=(pylambda/q)$. I am pretty sure this has no closed form.
    $endgroup$
    – Mike Earnest
    Dec 16 '18 at 19:24
















4












$begingroup$


Setup:



Consider the random variable $Y_N$ derived from
$$Y_N = sum_{i=0}^N X_i$$
where $N$ is a random variable with distribution $p_n = {M choose n} p^n q^{M-n}$ (binomial i.e. $M$ Bernoulli trials) and the $X_i$ are random variables with distribution $f(x) = lambda e^{-lambda x}$ (exponential).
I'd like to get the pdf of $y$.
Previously I asked this question which led me to expect the distribution is the sum
$$p(y) = sum_{i=1}^M {M choose i} p^i q^{M-i} frac{lambda^iy^{i-1}e^{-lambda y}}{(i-1)!} $$
or the integral (inverse Fourier transform of the characteristic function)
$$p(y) = int_{-infty}^{infty} frac{dk}{2pi} e^{-iky}(frac{plambda}{lambda-ik}+q)^M = int_{-infty}^{infty} frac{dk}{2pi}e^{-iky}(frac{ilambda + kq}{ilambda + k})^M$$
The second equality follows from $p+q=1$.
I'd like to solve either one of these problems in closed form if possible. The moment generating function these derive from is
$$ p(s) = (frac{plambda}{lambda-s}+q)^M.$$
The characteristic function is $p(s=ik)$.



Question:



What is the distribution $p(y)$ of $Y_N$?



Attempts:



I tried binomial expanding $p(s)$ in the fourier transform form, which leads to a sum over integrals like $int_{-infty}^{infty} du u^{-m}e^{-a u}$ where $m$ is a positive integer -- almost a sum of gamma functions but with negative parameters and over the wrong limits.



I tried manipulaing the sum form into something I could use the binomial formula on, but I can't obtain binomial coefficients from the factorial prefactors in the sum.



Any thoughts? I had thought a compound binomial-exponential process would have been well studied with a closed form solution but I couldn't find anything immediately relevant.



Other:



This paper explores a very similar problem but I'll have to read carefully to see how it relates (link).










share|cite|improve this question











$endgroup$








  • 1




    $begingroup$
    Your definition of the random number is a little confusing is it N or (as it appears in usage) n or i?
    $endgroup$
    – herb steinberg
    Dec 14 '18 at 22:24










  • $begingroup$
    $M$ is a parameter of the binomial distribution (it was $N$ before), i.e. the number of Bernoulli trials of probabilities $p$ and $q$ which contribute to the distribution. $N$ is a random variable, and $n$ is a particular realization of that random variable. Edited thanks @MikeEarnest
    $endgroup$
    – kevinkayaks
    Dec 14 '18 at 22:31










  • $begingroup$
    The question has no answer without any independence assumptions.
    $endgroup$
    – Kavi Rama Murthy
    Dec 14 '18 at 23:28










  • $begingroup$
    The $X_i$ are all independent and identically distributed. $X_i$ and $N$ are independent. These assumptions are embedded into the two expressions I gave for $p(y)$.
    $endgroup$
    – kevinkayaks
    Dec 14 '18 at 23:35








  • 1




    $begingroup$
    Your first expression for $p(y)$ can be written $e^{-lambda y}frac{q^M}ysum_{i=0}^M binom{M}i frac{t^i}{(i-1)!}$, where $t=(pylambda/q)$. I am pretty sure this has no closed form.
    $endgroup$
    – Mike Earnest
    Dec 16 '18 at 19:24














4












4








4





$begingroup$


Setup:



Consider the random variable $Y_N$ derived from
$$Y_N = sum_{i=0}^N X_i$$
where $N$ is a random variable with distribution $p_n = {M choose n} p^n q^{M-n}$ (binomial i.e. $M$ Bernoulli trials) and the $X_i$ are random variables with distribution $f(x) = lambda e^{-lambda x}$ (exponential).
I'd like to get the pdf of $y$.
Previously I asked this question which led me to expect the distribution is the sum
$$p(y) = sum_{i=1}^M {M choose i} p^i q^{M-i} frac{lambda^iy^{i-1}e^{-lambda y}}{(i-1)!} $$
or the integral (inverse Fourier transform of the characteristic function)
$$p(y) = int_{-infty}^{infty} frac{dk}{2pi} e^{-iky}(frac{plambda}{lambda-ik}+q)^M = int_{-infty}^{infty} frac{dk}{2pi}e^{-iky}(frac{ilambda + kq}{ilambda + k})^M$$
The second equality follows from $p+q=1$.
I'd like to solve either one of these problems in closed form if possible. The moment generating function these derive from is
$$ p(s) = (frac{plambda}{lambda-s}+q)^M.$$
The characteristic function is $p(s=ik)$.



Question:



What is the distribution $p(y)$ of $Y_N$?



Attempts:



I tried binomial expanding $p(s)$ in the fourier transform form, which leads to a sum over integrals like $int_{-infty}^{infty} du u^{-m}e^{-a u}$ where $m$ is a positive integer -- almost a sum of gamma functions but with negative parameters and over the wrong limits.



I tried manipulaing the sum form into something I could use the binomial formula on, but I can't obtain binomial coefficients from the factorial prefactors in the sum.



Any thoughts? I had thought a compound binomial-exponential process would have been well studied with a closed form solution but I couldn't find anything immediately relevant.



Other:



This paper explores a very similar problem but I'll have to read carefully to see how it relates (link).










share|cite|improve this question











$endgroup$




Setup:



Consider the random variable $Y_N$ derived from
$$Y_N = sum_{i=0}^N X_i$$
where $N$ is a random variable with distribution $p_n = {M choose n} p^n q^{M-n}$ (binomial i.e. $M$ Bernoulli trials) and the $X_i$ are random variables with distribution $f(x) = lambda e^{-lambda x}$ (exponential).
I'd like to get the pdf of $y$.
Previously I asked this question which led me to expect the distribution is the sum
$$p(y) = sum_{i=1}^M {M choose i} p^i q^{M-i} frac{lambda^iy^{i-1}e^{-lambda y}}{(i-1)!} $$
or the integral (inverse Fourier transform of the characteristic function)
$$p(y) = int_{-infty}^{infty} frac{dk}{2pi} e^{-iky}(frac{plambda}{lambda-ik}+q)^M = int_{-infty}^{infty} frac{dk}{2pi}e^{-iky}(frac{ilambda + kq}{ilambda + k})^M$$
The second equality follows from $p+q=1$.
I'd like to solve either one of these problems in closed form if possible. The moment generating function these derive from is
$$ p(s) = (frac{plambda}{lambda-s}+q)^M.$$
The characteristic function is $p(s=ik)$.



Question:



What is the distribution $p(y)$ of $Y_N$?



Attempts:



I tried binomial expanding $p(s)$ in the fourier transform form, which leads to a sum over integrals like $int_{-infty}^{infty} du u^{-m}e^{-a u}$ where $m$ is a positive integer -- almost a sum of gamma functions but with negative parameters and over the wrong limits.



I tried manipulaing the sum form into something I could use the binomial formula on, but I can't obtain binomial coefficients from the factorial prefactors in the sum.



Any thoughts? I had thought a compound binomial-exponential process would have been well studied with a closed form solution but I couldn't find anything immediately relevant.



Other:



This paper explores a very similar problem but I'll have to read carefully to see how it relates (link).







integration special-functions closed-form fourier-transform






share|cite|improve this question















share|cite|improve this question













share|cite|improve this question




share|cite|improve this question








edited Dec 15 '18 at 21:30







kevinkayaks

















asked Dec 14 '18 at 22:18









kevinkayakskevinkayaks

1558




1558








  • 1




    $begingroup$
    Your definition of the random number is a little confusing is it N or (as it appears in usage) n or i?
    $endgroup$
    – herb steinberg
    Dec 14 '18 at 22:24










  • $begingroup$
    $M$ is a parameter of the binomial distribution (it was $N$ before), i.e. the number of Bernoulli trials of probabilities $p$ and $q$ which contribute to the distribution. $N$ is a random variable, and $n$ is a particular realization of that random variable. Edited thanks @MikeEarnest
    $endgroup$
    – kevinkayaks
    Dec 14 '18 at 22:31










  • $begingroup$
    The question has no answer without any independence assumptions.
    $endgroup$
    – Kavi Rama Murthy
    Dec 14 '18 at 23:28










  • $begingroup$
    The $X_i$ are all independent and identically distributed. $X_i$ and $N$ are independent. These assumptions are embedded into the two expressions I gave for $p(y)$.
    $endgroup$
    – kevinkayaks
    Dec 14 '18 at 23:35








  • 1




    $begingroup$
    Your first expression for $p(y)$ can be written $e^{-lambda y}frac{q^M}ysum_{i=0}^M binom{M}i frac{t^i}{(i-1)!}$, where $t=(pylambda/q)$. I am pretty sure this has no closed form.
    $endgroup$
    – Mike Earnest
    Dec 16 '18 at 19:24














  • 1




    $begingroup$
    Your definition of the random number is a little confusing is it N or (as it appears in usage) n or i?
    $endgroup$
    – herb steinberg
    Dec 14 '18 at 22:24










  • $begingroup$
    $M$ is a parameter of the binomial distribution (it was $N$ before), i.e. the number of Bernoulli trials of probabilities $p$ and $q$ which contribute to the distribution. $N$ is a random variable, and $n$ is a particular realization of that random variable. Edited thanks @MikeEarnest
    $endgroup$
    – kevinkayaks
    Dec 14 '18 at 22:31










  • $begingroup$
    The question has no answer without any independence assumptions.
    $endgroup$
    – Kavi Rama Murthy
    Dec 14 '18 at 23:28










  • $begingroup$
    The $X_i$ are all independent and identically distributed. $X_i$ and $N$ are independent. These assumptions are embedded into the two expressions I gave for $p(y)$.
    $endgroup$
    – kevinkayaks
    Dec 14 '18 at 23:35








  • 1




    $begingroup$
    Your first expression for $p(y)$ can be written $e^{-lambda y}frac{q^M}ysum_{i=0}^M binom{M}i frac{t^i}{(i-1)!}$, where $t=(pylambda/q)$. I am pretty sure this has no closed form.
    $endgroup$
    – Mike Earnest
    Dec 16 '18 at 19:24








1




1




$begingroup$
Your definition of the random number is a little confusing is it N or (as it appears in usage) n or i?
$endgroup$
– herb steinberg
Dec 14 '18 at 22:24




$begingroup$
Your definition of the random number is a little confusing is it N or (as it appears in usage) n or i?
$endgroup$
– herb steinberg
Dec 14 '18 at 22:24












$begingroup$
$M$ is a parameter of the binomial distribution (it was $N$ before), i.e. the number of Bernoulli trials of probabilities $p$ and $q$ which contribute to the distribution. $N$ is a random variable, and $n$ is a particular realization of that random variable. Edited thanks @MikeEarnest
$endgroup$
– kevinkayaks
Dec 14 '18 at 22:31




$begingroup$
$M$ is a parameter of the binomial distribution (it was $N$ before), i.e. the number of Bernoulli trials of probabilities $p$ and $q$ which contribute to the distribution. $N$ is a random variable, and $n$ is a particular realization of that random variable. Edited thanks @MikeEarnest
$endgroup$
– kevinkayaks
Dec 14 '18 at 22:31












$begingroup$
The question has no answer without any independence assumptions.
$endgroup$
– Kavi Rama Murthy
Dec 14 '18 at 23:28




$begingroup$
The question has no answer without any independence assumptions.
$endgroup$
– Kavi Rama Murthy
Dec 14 '18 at 23:28












$begingroup$
The $X_i$ are all independent and identically distributed. $X_i$ and $N$ are independent. These assumptions are embedded into the two expressions I gave for $p(y)$.
$endgroup$
– kevinkayaks
Dec 14 '18 at 23:35






$begingroup$
The $X_i$ are all independent and identically distributed. $X_i$ and $N$ are independent. These assumptions are embedded into the two expressions I gave for $p(y)$.
$endgroup$
– kevinkayaks
Dec 14 '18 at 23:35






1




1




$begingroup$
Your first expression for $p(y)$ can be written $e^{-lambda y}frac{q^M}ysum_{i=0}^M binom{M}i frac{t^i}{(i-1)!}$, where $t=(pylambda/q)$. I am pretty sure this has no closed form.
$endgroup$
– Mike Earnest
Dec 16 '18 at 19:24




$begingroup$
Your first expression for $p(y)$ can be written $e^{-lambda y}frac{q^M}ysum_{i=0}^M binom{M}i frac{t^i}{(i-1)!}$, where $t=(pylambda/q)$. I am pretty sure this has no closed form.
$endgroup$
– Mike Earnest
Dec 16 '18 at 19:24










1 Answer
1






active

oldest

votes


















2












$begingroup$

I was able to compute the integral you presented using contour integration in the complex plane.



For the case of $lambda > 0$, $y <0$, it is easy to show via contour integration that



$$int_{-infty}^infty dfrac{dk}{2pi}e^{-iyk}left(dfrac{ilambda p}{k - (-ilambda)}+qright)^M = 0$$



For the more interesting case of $lambda > 0$, $y > 0$, one can show via contour integration, and algebraic manipulations to generate a Laurent Series, that



$$begin{align*}int_{-infty}^infty dfrac{dk}{2pi}e^{-iyk}left(dfrac{ilambda p}{k - (-ilambda)}+qright)^M &= -2pi i space mathrm{Res}_{z=-ilambda}left[dfrac{1}{2pi}e^{-iyz}left(dfrac{ilambda p}{z - (-ilambda)}+qright)^Mright]\
\
&= -2pi i space mathrm{Res}_{z=-ilambda}left[dfrac{1}{2pi}e^{-iyz}sum_{j=0}^{M}binom{M}{j}left(dfrac{ilambda p}{z - (-ilambda)}right)^jq^{M-j}right]\
\
&= -2pi i space mathrm{Res}_{z=-ilambda}left[dfrac{1}{2pi}left(sum_{n=0}^infty dfrac{left(-iyright)^n e^{-lambda y}}{n!}left(z-left(-ilambdaright)right)^nright)left(sum_{j=0}^{M}binom{M}{j}left(dfrac{ilambda p}{z - (-ilambda)}right)^jq^{M-j}right)right]\
\
&= -2pi i dfrac{1}{2pi}sum_{j=1}^M dfrac{left(-iyright)^{j-1}e^{-lambda y}}{left(j-1right)!}binom{M}{j}left(ilambda pright)^jq^{M-j} \
\
&= sum_{j=1}^M binom{M}{j} p^j q^{M-j}dfrac{lambda^j y^{j-1} e^{-lambda y}}{left(j-1right)!}\
\
&mbox{(confirmation of exactly what you already thought to be true)}\
\
&= dfrac{q^M e^{-lambda y}}{y}sum_{j=1}^M binom{M}{j} left(dfrac{lambda y p}{q}right)^j dfrac{1}{left(j-1right)!}\
end{align*}$$



For the case of $lambda > 0$, $y =0$, the integral diverges due to the "$+q$". The integral diverges at this single value of $y$ with a weight of $q^M$, given some hindsight with the convolution theorem and the definition of a CDF.



The integral really does not provide any new information regarding the series. You need to manipulate the series form into something you find palatable.



The above development of the Laurent Series, to find the residue, does provide some insights into how portions of each term in the final sum arise. The binomial coefficients come from the binomial expansion, as you would expect. The factorial factors come from the Taylor Series expansion of the exponential function. The final summation comes from multiplying series terms together and adding to find the coefficient of the $(z-(-ilambda))^{-1}$ term of the Laurent Series.



Consolidating the results for all real values of $y$, the expression should really be written as:



$$begin{align*}p(y) &= q^Mdelta(y)+H(y)dfrac{q^M e^{-lambda y}}{y}sum_{j=1}^M binom{M}{j} left(dfrac{lambda y p}{q}right)^j dfrac{1}{left(j-1right)!}\
\
&= q^Mdelta(y)+ H(y) dfrac{p}{q} q^{M} lambda e^{-lambda y}sum_{j=0}^{M-1} binom{M}{j+1} left(dfrac{p}{q}lambda yright)^{j} dfrac{1}{j!}\
end{align*}$$



Where $H(y)$ is the Heaviside unit step and $delta(y)$ is the Dirac Delta function.



Using this expression, convolving the $M=1$ case with itself $M-1$ times, yields the correct expression for the $M^{th}$ case, as expected given the convolution theorem of the Fourier Transform.



Using the above expression for $p(y)$, we find



$$int_{-infty}^infty p(y) dy = (q+p)^M =1$$



which is what one would expect.



To replace the series, Wolfram Alpha comes up with this "closed" form in terms of a hypergeometric function:



$$begin{align*}p(y) &= q^Mdelta(y)+H(y) dfrac{p}{q} q^{M} lambda e^{-lambda y}sum_{j=0}^{M-1} binom{M}{j+1} left(dfrac{p}{q}lambda y right)^{j} dfrac{1}{j!}\
\
&= q^Mdelta(y)+ H(y) dfrac{p}{q} q^{M} lambda e^{-lambda y} M space {}_1F_1 left(1-M;2;-dfrac{p}{q}lambda yright)
end{align*}$$



Update to derive the CDF from the PDF



To derive the CDF, let's rewrite the PDF a little bit. I'll be using the following expression involving the Incomplete Upper Gamma Function in the process



$$dfrac{partial}{partial y}Gamma(j,lambda y) =dfrac{partial}{partial (lambda y)}Gamma(j,lambda y) cdot dfrac{partial}{partial y}lambda y = -(lambda y)^{j-1}e^{-lambda y}lambda$$



So working the PDF a bit



$$begin{align*}p(y) &= q^Mdelta(y)+H(y) dfrac{p}{q} q^{M} lambda e^{-lambda y}sum_{j=0}^{M-1} binom{M}{j+1} left(dfrac{p}{q}lambda y right)^{j} dfrac{1}{j!}\
\
&= delta(y)q^M+H(y) lambda e^{-lambda y}sum_{j=0}^{M-1} binom{M}{j+1} q^{M-(j+1)}p^{j+1}left(lambda y right)^{j} dfrac{1}{j!}\
\
&= delta(y)q^M+H(y) lambda e^{-lambda y}sum_{j=1}^{M} binom{M}{j} q^{M-j}p^{j}dfrac{1}{(j-1)!}left(lambda y right)^{j-1} \
\
&= delta(y)q^M-H(y) sum_{j=1}^{M} binom{M}{j} q^{M-j}p^{j}dfrac{1}{(j-1)!}(-1)left(lambda y right)^{j-1} e^{-lambda y}lambda \
\
&= delta(y)q^M-H(y) sum_{j=1}^{M} binom{M}{j} q^{M-j}p^{j}dfrac{partial}{partial y}dfrac{Gamma(j,lambda y)}{Gamma(j)} \
end{align*}$$



To integrate the PDF to get the CDF, I'll be using the following relations:



$$int_{-infty}^{x} H(t)f'(t) dt = H(x)int_0^x f'(t) dt = H(x)left[f(x) - f(0)right]$$



$$int_{-infty}^x delta(t) f(t)dt = H(x)f(0)$$



So, integrating the PDF to find the CDF:



$$begin{align*}P(y) &= int_{-infty}^y p(t) dt\
\
&= int_{-infty}^y delta(t)q^M dt - sum_{j=1}^{M} binom{M}{j} q^{M-j}p^{j}int_{-infty}^y H(t)dfrac{partial}{partial t}dfrac{Gamma(j,lambda t)}{Gamma(j)} dt\
\
&= H(y)q^M -sum_{j=1}^{M} binom{M}{j} q^{M-j}p^{j} H(y)left[dfrac{Gamma(j,lambda y)}{Gamma(j)}-dfrac{Gamma(j,0)}{Gamma(j)}right]\
\
&= H(y)left[q^M -sum_{j=1}^{M} binom{M}{j} q^{M-j}p^{j} left(dfrac{Gamma(j,lambda y)}{Gamma(j)}-1right)right]\
\
&= H(y)left[q^M +sum_{j=1}^{M} binom{M}{j} q^{M-j}p^{j} -sum_{j=1}^{M} binom{M}{j} q^{M-j}p^{j}dfrac{Gamma(j,lambda y)}{Gamma(j)}right]\
\
&= H(y)left[(q+p)^M -sum_{j=1}^{M} binom{M}{j} q^{M-j}p^{j}dfrac{Gamma(j,lambda y)}{Gamma(j)}right]\
\
P(y) &= H(y)left[1 -sum_{j=1}^{M} binom{M}{j} q^{M-j}p^{j}dfrac{Gamma(j,lambda y)}{Gamma(j)}right]\
end{align*}$$






share|cite|improve this answer











$endgroup$













  • $begingroup$
    Solid @Andy. I especially appreciate the contour integration to show my two expressions are equivalent. That's satisfying. I guess the hypergeometric solution is as good as it gets. I was hoping for something simpler for the application which brought this problem up, but it is what it is.
    $endgroup$
    – kevinkayaks
    Dec 22 '18 at 2:36










  • $begingroup$
    Thanks. I just added an update to rework the PDF a little, so I could also derive the CDF.
    $endgroup$
    – Andy Walls
    Dec 23 '18 at 17:55










  • $begingroup$
    Yeah, there seems to be no easy way to simplify those $Gamma(,)$ related weights from the otherwise binomial expansion.
    $endgroup$
    – Andy Walls
    Dec 23 '18 at 18:01











Your Answer





StackExchange.ifUsing("editor", function () {
return StackExchange.using("mathjaxEditing", function () {
StackExchange.MarkdownEditor.creationCallbacks.add(function (editor, postfix) {
StackExchange.mathjaxEditing.prepareWmdForMathJax(editor, postfix, [["$", "$"], ["\\(","\\)"]]);
});
});
}, "mathjax-editing");

StackExchange.ready(function() {
var channelOptions = {
tags: "".split(" "),
id: "69"
};
initTagRenderer("".split(" "), "".split(" "), channelOptions);

StackExchange.using("externalEditor", function() {
// Have to fire editor after snippets, if snippets enabled
if (StackExchange.settings.snippets.snippetsEnabled) {
StackExchange.using("snippets", function() {
createEditor();
});
}
else {
createEditor();
}
});

function createEditor() {
StackExchange.prepareEditor({
heartbeatType: 'answer',
autoActivateHeartbeat: false,
convertImagesToLinks: true,
noModals: true,
showLowRepImageUploadWarning: true,
reputationToPostImages: 10,
bindNavPrevention: true,
postfix: "",
imageUploader: {
brandingHtml: "Powered by u003ca class="icon-imgur-white" href="https://imgur.com/"u003eu003c/au003e",
contentPolicyHtml: "User contributions licensed under u003ca href="https://creativecommons.org/licenses/by-sa/3.0/"u003ecc by-sa 3.0 with attribution requiredu003c/au003e u003ca href="https://stackoverflow.com/legal/content-policy"u003e(content policy)u003c/au003e",
allowUrls: true
},
noCode: true, onDemand: true,
discardSelector: ".discard-answer"
,immediatelyShowMarkdownHelp:true
});


}
});














draft saved

draft discarded


















StackExchange.ready(
function () {
StackExchange.openid.initPostLogin('.new-post-login', 'https%3a%2f%2fmath.stackexchange.com%2fquestions%2f3039945%2fcompound-binomial-exponential-closed-form-for-the-pdf%23new-answer', 'question_page');
}
);

Post as a guest















Required, but never shown

























1 Answer
1






active

oldest

votes








1 Answer
1






active

oldest

votes









active

oldest

votes






active

oldest

votes









2












$begingroup$

I was able to compute the integral you presented using contour integration in the complex plane.



For the case of $lambda > 0$, $y <0$, it is easy to show via contour integration that



$$int_{-infty}^infty dfrac{dk}{2pi}e^{-iyk}left(dfrac{ilambda p}{k - (-ilambda)}+qright)^M = 0$$



For the more interesting case of $lambda > 0$, $y > 0$, one can show via contour integration, and algebraic manipulations to generate a Laurent Series, that



$$begin{align*}int_{-infty}^infty dfrac{dk}{2pi}e^{-iyk}left(dfrac{ilambda p}{k - (-ilambda)}+qright)^M &= -2pi i space mathrm{Res}_{z=-ilambda}left[dfrac{1}{2pi}e^{-iyz}left(dfrac{ilambda p}{z - (-ilambda)}+qright)^Mright]\
\
&= -2pi i space mathrm{Res}_{z=-ilambda}left[dfrac{1}{2pi}e^{-iyz}sum_{j=0}^{M}binom{M}{j}left(dfrac{ilambda p}{z - (-ilambda)}right)^jq^{M-j}right]\
\
&= -2pi i space mathrm{Res}_{z=-ilambda}left[dfrac{1}{2pi}left(sum_{n=0}^infty dfrac{left(-iyright)^n e^{-lambda y}}{n!}left(z-left(-ilambdaright)right)^nright)left(sum_{j=0}^{M}binom{M}{j}left(dfrac{ilambda p}{z - (-ilambda)}right)^jq^{M-j}right)right]\
\
&= -2pi i dfrac{1}{2pi}sum_{j=1}^M dfrac{left(-iyright)^{j-1}e^{-lambda y}}{left(j-1right)!}binom{M}{j}left(ilambda pright)^jq^{M-j} \
\
&= sum_{j=1}^M binom{M}{j} p^j q^{M-j}dfrac{lambda^j y^{j-1} e^{-lambda y}}{left(j-1right)!}\
\
&mbox{(confirmation of exactly what you already thought to be true)}\
\
&= dfrac{q^M e^{-lambda y}}{y}sum_{j=1}^M binom{M}{j} left(dfrac{lambda y p}{q}right)^j dfrac{1}{left(j-1right)!}\
end{align*}$$



For the case of $lambda > 0$, $y =0$, the integral diverges due to the "$+q$". The integral diverges at this single value of $y$ with a weight of $q^M$, given some hindsight with the convolution theorem and the definition of a CDF.



The integral really does not provide any new information regarding the series. You need to manipulate the series form into something you find palatable.



The above development of the Laurent Series, to find the residue, does provide some insights into how portions of each term in the final sum arise. The binomial coefficients come from the binomial expansion, as you would expect. The factorial factors come from the Taylor Series expansion of the exponential function. The final summation comes from multiplying series terms together and adding to find the coefficient of the $(z-(-ilambda))^{-1}$ term of the Laurent Series.



Consolidating the results for all real values of $y$, the expression should really be written as:



$$begin{align*}p(y) &= q^Mdelta(y)+H(y)dfrac{q^M e^{-lambda y}}{y}sum_{j=1}^M binom{M}{j} left(dfrac{lambda y p}{q}right)^j dfrac{1}{left(j-1right)!}\
\
&= q^Mdelta(y)+ H(y) dfrac{p}{q} q^{M} lambda e^{-lambda y}sum_{j=0}^{M-1} binom{M}{j+1} left(dfrac{p}{q}lambda yright)^{j} dfrac{1}{j!}\
end{align*}$$



Where $H(y)$ is the Heaviside unit step and $delta(y)$ is the Dirac Delta function.



Using this expression, convolving the $M=1$ case with itself $M-1$ times, yields the correct expression for the $M^{th}$ case, as expected given the convolution theorem of the Fourier Transform.



Using the above expression for $p(y)$, we find



$$int_{-infty}^infty p(y) dy = (q+p)^M =1$$



which is what one would expect.



To replace the series, Wolfram Alpha comes up with this "closed" form in terms of a hypergeometric function:



$$begin{align*}p(y) &= q^Mdelta(y)+H(y) dfrac{p}{q} q^{M} lambda e^{-lambda y}sum_{j=0}^{M-1} binom{M}{j+1} left(dfrac{p}{q}lambda y right)^{j} dfrac{1}{j!}\
\
&= q^Mdelta(y)+ H(y) dfrac{p}{q} q^{M} lambda e^{-lambda y} M space {}_1F_1 left(1-M;2;-dfrac{p}{q}lambda yright)
end{align*}$$



Update to derive the CDF from the PDF



To derive the CDF, let's rewrite the PDF a little bit. I'll be using the following expression involving the Incomplete Upper Gamma Function in the process



$$dfrac{partial}{partial y}Gamma(j,lambda y) =dfrac{partial}{partial (lambda y)}Gamma(j,lambda y) cdot dfrac{partial}{partial y}lambda y = -(lambda y)^{j-1}e^{-lambda y}lambda$$



So working the PDF a bit



$$begin{align*}p(y) &= q^Mdelta(y)+H(y) dfrac{p}{q} q^{M} lambda e^{-lambda y}sum_{j=0}^{M-1} binom{M}{j+1} left(dfrac{p}{q}lambda y right)^{j} dfrac{1}{j!}\
\
&= delta(y)q^M+H(y) lambda e^{-lambda y}sum_{j=0}^{M-1} binom{M}{j+1} q^{M-(j+1)}p^{j+1}left(lambda y right)^{j} dfrac{1}{j!}\
\
&= delta(y)q^M+H(y) lambda e^{-lambda y}sum_{j=1}^{M} binom{M}{j} q^{M-j}p^{j}dfrac{1}{(j-1)!}left(lambda y right)^{j-1} \
\
&= delta(y)q^M-H(y) sum_{j=1}^{M} binom{M}{j} q^{M-j}p^{j}dfrac{1}{(j-1)!}(-1)left(lambda y right)^{j-1} e^{-lambda y}lambda \
\
&= delta(y)q^M-H(y) sum_{j=1}^{M} binom{M}{j} q^{M-j}p^{j}dfrac{partial}{partial y}dfrac{Gamma(j,lambda y)}{Gamma(j)} \
end{align*}$$



To integrate the PDF to get the CDF, I'll be using the following relations:



$$int_{-infty}^{x} H(t)f'(t) dt = H(x)int_0^x f'(t) dt = H(x)left[f(x) - f(0)right]$$



$$int_{-infty}^x delta(t) f(t)dt = H(x)f(0)$$



So, integrating the PDF to find the CDF:



$$begin{align*}P(y) &= int_{-infty}^y p(t) dt\
\
&= int_{-infty}^y delta(t)q^M dt - sum_{j=1}^{M} binom{M}{j} q^{M-j}p^{j}int_{-infty}^y H(t)dfrac{partial}{partial t}dfrac{Gamma(j,lambda t)}{Gamma(j)} dt\
\
&= H(y)q^M -sum_{j=1}^{M} binom{M}{j} q^{M-j}p^{j} H(y)left[dfrac{Gamma(j,lambda y)}{Gamma(j)}-dfrac{Gamma(j,0)}{Gamma(j)}right]\
\
&= H(y)left[q^M -sum_{j=1}^{M} binom{M}{j} q^{M-j}p^{j} left(dfrac{Gamma(j,lambda y)}{Gamma(j)}-1right)right]\
\
&= H(y)left[q^M +sum_{j=1}^{M} binom{M}{j} q^{M-j}p^{j} -sum_{j=1}^{M} binom{M}{j} q^{M-j}p^{j}dfrac{Gamma(j,lambda y)}{Gamma(j)}right]\
\
&= H(y)left[(q+p)^M -sum_{j=1}^{M} binom{M}{j} q^{M-j}p^{j}dfrac{Gamma(j,lambda y)}{Gamma(j)}right]\
\
P(y) &= H(y)left[1 -sum_{j=1}^{M} binom{M}{j} q^{M-j}p^{j}dfrac{Gamma(j,lambda y)}{Gamma(j)}right]\
end{align*}$$






share|cite|improve this answer











$endgroup$













  • $begingroup$
    Solid @Andy. I especially appreciate the contour integration to show my two expressions are equivalent. That's satisfying. I guess the hypergeometric solution is as good as it gets. I was hoping for something simpler for the application which brought this problem up, but it is what it is.
    $endgroup$
    – kevinkayaks
    Dec 22 '18 at 2:36










  • $begingroup$
    Thanks. I just added an update to rework the PDF a little, so I could also derive the CDF.
    $endgroup$
    – Andy Walls
    Dec 23 '18 at 17:55










  • $begingroup$
    Yeah, there seems to be no easy way to simplify those $Gamma(,)$ related weights from the otherwise binomial expansion.
    $endgroup$
    – Andy Walls
    Dec 23 '18 at 18:01
















2












$begingroup$

I was able to compute the integral you presented using contour integration in the complex plane.



For the case of $lambda > 0$, $y <0$, it is easy to show via contour integration that



$$int_{-infty}^infty dfrac{dk}{2pi}e^{-iyk}left(dfrac{ilambda p}{k - (-ilambda)}+qright)^M = 0$$



For the more interesting case of $lambda > 0$, $y > 0$, one can show via contour integration, and algebraic manipulations to generate a Laurent Series, that



$$begin{align*}int_{-infty}^infty dfrac{dk}{2pi}e^{-iyk}left(dfrac{ilambda p}{k - (-ilambda)}+qright)^M &= -2pi i space mathrm{Res}_{z=-ilambda}left[dfrac{1}{2pi}e^{-iyz}left(dfrac{ilambda p}{z - (-ilambda)}+qright)^Mright]\
\
&= -2pi i space mathrm{Res}_{z=-ilambda}left[dfrac{1}{2pi}e^{-iyz}sum_{j=0}^{M}binom{M}{j}left(dfrac{ilambda p}{z - (-ilambda)}right)^jq^{M-j}right]\
\
&= -2pi i space mathrm{Res}_{z=-ilambda}left[dfrac{1}{2pi}left(sum_{n=0}^infty dfrac{left(-iyright)^n e^{-lambda y}}{n!}left(z-left(-ilambdaright)right)^nright)left(sum_{j=0}^{M}binom{M}{j}left(dfrac{ilambda p}{z - (-ilambda)}right)^jq^{M-j}right)right]\
\
&= -2pi i dfrac{1}{2pi}sum_{j=1}^M dfrac{left(-iyright)^{j-1}e^{-lambda y}}{left(j-1right)!}binom{M}{j}left(ilambda pright)^jq^{M-j} \
\
&= sum_{j=1}^M binom{M}{j} p^j q^{M-j}dfrac{lambda^j y^{j-1} e^{-lambda y}}{left(j-1right)!}\
\
&mbox{(confirmation of exactly what you already thought to be true)}\
\
&= dfrac{q^M e^{-lambda y}}{y}sum_{j=1}^M binom{M}{j} left(dfrac{lambda y p}{q}right)^j dfrac{1}{left(j-1right)!}\
end{align*}$$



For the case of $lambda > 0$, $y =0$, the integral diverges due to the "$+q$". The integral diverges at this single value of $y$ with a weight of $q^M$, given some hindsight with the convolution theorem and the definition of a CDF.



The integral really does not provide any new information regarding the series. You need to manipulate the series form into something you find palatable.



The above development of the Laurent Series, to find the residue, does provide some insights into how portions of each term in the final sum arise. The binomial coefficients come from the binomial expansion, as you would expect. The factorial factors come from the Taylor Series expansion of the exponential function. The final summation comes from multiplying series terms together and adding to find the coefficient of the $(z-(-ilambda))^{-1}$ term of the Laurent Series.



Consolidating the results for all real values of $y$, the expression should really be written as:



$$begin{align*}p(y) &= q^Mdelta(y)+H(y)dfrac{q^M e^{-lambda y}}{y}sum_{j=1}^M binom{M}{j} left(dfrac{lambda y p}{q}right)^j dfrac{1}{left(j-1right)!}\
\
&= q^Mdelta(y)+ H(y) dfrac{p}{q} q^{M} lambda e^{-lambda y}sum_{j=0}^{M-1} binom{M}{j+1} left(dfrac{p}{q}lambda yright)^{j} dfrac{1}{j!}\
end{align*}$$



Where $H(y)$ is the Heaviside unit step and $delta(y)$ is the Dirac Delta function.



Using this expression, convolving the $M=1$ case with itself $M-1$ times, yields the correct expression for the $M^{th}$ case, as expected given the convolution theorem of the Fourier Transform.



Using the above expression for $p(y)$, we find



$$int_{-infty}^infty p(y) dy = (q+p)^M =1$$



which is what one would expect.



To replace the series, Wolfram Alpha comes up with this "closed" form in terms of a hypergeometric function:



$$begin{align*}p(y) &= q^Mdelta(y)+H(y) dfrac{p}{q} q^{M} lambda e^{-lambda y}sum_{j=0}^{M-1} binom{M}{j+1} left(dfrac{p}{q}lambda y right)^{j} dfrac{1}{j!}\
\
&= q^Mdelta(y)+ H(y) dfrac{p}{q} q^{M} lambda e^{-lambda y} M space {}_1F_1 left(1-M;2;-dfrac{p}{q}lambda yright)
end{align*}$$



Update to derive the CDF from the PDF



To derive the CDF, let's rewrite the PDF a little bit. I'll be using the following expression involving the Incomplete Upper Gamma Function in the process



$$dfrac{partial}{partial y}Gamma(j,lambda y) =dfrac{partial}{partial (lambda y)}Gamma(j,lambda y) cdot dfrac{partial}{partial y}lambda y = -(lambda y)^{j-1}e^{-lambda y}lambda$$



So working the PDF a bit



$$begin{align*}p(y) &= q^Mdelta(y)+H(y) dfrac{p}{q} q^{M} lambda e^{-lambda y}sum_{j=0}^{M-1} binom{M}{j+1} left(dfrac{p}{q}lambda y right)^{j} dfrac{1}{j!}\
\
&= delta(y)q^M+H(y) lambda e^{-lambda y}sum_{j=0}^{M-1} binom{M}{j+1} q^{M-(j+1)}p^{j+1}left(lambda y right)^{j} dfrac{1}{j!}\
\
&= delta(y)q^M+H(y) lambda e^{-lambda y}sum_{j=1}^{M} binom{M}{j} q^{M-j}p^{j}dfrac{1}{(j-1)!}left(lambda y right)^{j-1} \
\
&= delta(y)q^M-H(y) sum_{j=1}^{M} binom{M}{j} q^{M-j}p^{j}dfrac{1}{(j-1)!}(-1)left(lambda y right)^{j-1} e^{-lambda y}lambda \
\
&= delta(y)q^M-H(y) sum_{j=1}^{M} binom{M}{j} q^{M-j}p^{j}dfrac{partial}{partial y}dfrac{Gamma(j,lambda y)}{Gamma(j)} \
end{align*}$$



To integrate the PDF to get the CDF, I'll be using the following relations:



$$int_{-infty}^{x} H(t)f'(t) dt = H(x)int_0^x f'(t) dt = H(x)left[f(x) - f(0)right]$$



$$int_{-infty}^x delta(t) f(t)dt = H(x)f(0)$$



So, integrating the PDF to find the CDF:



$$begin{align*}P(y) &= int_{-infty}^y p(t) dt\
\
&= int_{-infty}^y delta(t)q^M dt - sum_{j=1}^{M} binom{M}{j} q^{M-j}p^{j}int_{-infty}^y H(t)dfrac{partial}{partial t}dfrac{Gamma(j,lambda t)}{Gamma(j)} dt\
\
&= H(y)q^M -sum_{j=1}^{M} binom{M}{j} q^{M-j}p^{j} H(y)left[dfrac{Gamma(j,lambda y)}{Gamma(j)}-dfrac{Gamma(j,0)}{Gamma(j)}right]\
\
&= H(y)left[q^M -sum_{j=1}^{M} binom{M}{j} q^{M-j}p^{j} left(dfrac{Gamma(j,lambda y)}{Gamma(j)}-1right)right]\
\
&= H(y)left[q^M +sum_{j=1}^{M} binom{M}{j} q^{M-j}p^{j} -sum_{j=1}^{M} binom{M}{j} q^{M-j}p^{j}dfrac{Gamma(j,lambda y)}{Gamma(j)}right]\
\
&= H(y)left[(q+p)^M -sum_{j=1}^{M} binom{M}{j} q^{M-j}p^{j}dfrac{Gamma(j,lambda y)}{Gamma(j)}right]\
\
P(y) &= H(y)left[1 -sum_{j=1}^{M} binom{M}{j} q^{M-j}p^{j}dfrac{Gamma(j,lambda y)}{Gamma(j)}right]\
end{align*}$$






share|cite|improve this answer











$endgroup$













  • $begingroup$
    Solid @Andy. I especially appreciate the contour integration to show my two expressions are equivalent. That's satisfying. I guess the hypergeometric solution is as good as it gets. I was hoping for something simpler for the application which brought this problem up, but it is what it is.
    $endgroup$
    – kevinkayaks
    Dec 22 '18 at 2:36










  • $begingroup$
    Thanks. I just added an update to rework the PDF a little, so I could also derive the CDF.
    $endgroup$
    – Andy Walls
    Dec 23 '18 at 17:55










  • $begingroup$
    Yeah, there seems to be no easy way to simplify those $Gamma(,)$ related weights from the otherwise binomial expansion.
    $endgroup$
    – Andy Walls
    Dec 23 '18 at 18:01














2












2








2





$begingroup$

I was able to compute the integral you presented using contour integration in the complex plane.



For the case of $lambda > 0$, $y <0$, it is easy to show via contour integration that



$$int_{-infty}^infty dfrac{dk}{2pi}e^{-iyk}left(dfrac{ilambda p}{k - (-ilambda)}+qright)^M = 0$$



For the more interesting case of $lambda > 0$, $y > 0$, one can show via contour integration, and algebraic manipulations to generate a Laurent Series, that



$$begin{align*}int_{-infty}^infty dfrac{dk}{2pi}e^{-iyk}left(dfrac{ilambda p}{k - (-ilambda)}+qright)^M &= -2pi i space mathrm{Res}_{z=-ilambda}left[dfrac{1}{2pi}e^{-iyz}left(dfrac{ilambda p}{z - (-ilambda)}+qright)^Mright]\
\
&= -2pi i space mathrm{Res}_{z=-ilambda}left[dfrac{1}{2pi}e^{-iyz}sum_{j=0}^{M}binom{M}{j}left(dfrac{ilambda p}{z - (-ilambda)}right)^jq^{M-j}right]\
\
&= -2pi i space mathrm{Res}_{z=-ilambda}left[dfrac{1}{2pi}left(sum_{n=0}^infty dfrac{left(-iyright)^n e^{-lambda y}}{n!}left(z-left(-ilambdaright)right)^nright)left(sum_{j=0}^{M}binom{M}{j}left(dfrac{ilambda p}{z - (-ilambda)}right)^jq^{M-j}right)right]\
\
&= -2pi i dfrac{1}{2pi}sum_{j=1}^M dfrac{left(-iyright)^{j-1}e^{-lambda y}}{left(j-1right)!}binom{M}{j}left(ilambda pright)^jq^{M-j} \
\
&= sum_{j=1}^M binom{M}{j} p^j q^{M-j}dfrac{lambda^j y^{j-1} e^{-lambda y}}{left(j-1right)!}\
\
&mbox{(confirmation of exactly what you already thought to be true)}\
\
&= dfrac{q^M e^{-lambda y}}{y}sum_{j=1}^M binom{M}{j} left(dfrac{lambda y p}{q}right)^j dfrac{1}{left(j-1right)!}\
end{align*}$$



For the case of $lambda > 0$, $y =0$, the integral diverges due to the "$+q$". The integral diverges at this single value of $y$ with a weight of $q^M$, given some hindsight with the convolution theorem and the definition of a CDF.



The integral really does not provide any new information regarding the series. You need to manipulate the series form into something you find palatable.



The above development of the Laurent Series, to find the residue, does provide some insights into how portions of each term in the final sum arise. The binomial coefficients come from the binomial expansion, as you would expect. The factorial factors come from the Taylor Series expansion of the exponential function. The final summation comes from multiplying series terms together and adding to find the coefficient of the $(z-(-ilambda))^{-1}$ term of the Laurent Series.



Consolidating the results for all real values of $y$, the expression should really be written as:



$$begin{align*}p(y) &= q^Mdelta(y)+H(y)dfrac{q^M e^{-lambda y}}{y}sum_{j=1}^M binom{M}{j} left(dfrac{lambda y p}{q}right)^j dfrac{1}{left(j-1right)!}\
\
&= q^Mdelta(y)+ H(y) dfrac{p}{q} q^{M} lambda e^{-lambda y}sum_{j=0}^{M-1} binom{M}{j+1} left(dfrac{p}{q}lambda yright)^{j} dfrac{1}{j!}\
end{align*}$$



Where $H(y)$ is the Heaviside unit step and $delta(y)$ is the Dirac Delta function.



Using this expression, convolving the $M=1$ case with itself $M-1$ times, yields the correct expression for the $M^{th}$ case, as expected given the convolution theorem of the Fourier Transform.



Using the above expression for $p(y)$, we find



$$int_{-infty}^infty p(y) dy = (q+p)^M =1$$



which is what one would expect.



To replace the series, Wolfram Alpha comes up with this "closed" form in terms of a hypergeometric function:



$$begin{align*}p(y) &= q^Mdelta(y)+H(y) dfrac{p}{q} q^{M} lambda e^{-lambda y}sum_{j=0}^{M-1} binom{M}{j+1} left(dfrac{p}{q}lambda y right)^{j} dfrac{1}{j!}\
\
&= q^Mdelta(y)+ H(y) dfrac{p}{q} q^{M} lambda e^{-lambda y} M space {}_1F_1 left(1-M;2;-dfrac{p}{q}lambda yright)
end{align*}$$



Update to derive the CDF from the PDF



To derive the CDF, let's rewrite the PDF a little bit. I'll be using the following expression involving the Incomplete Upper Gamma Function in the process



$$dfrac{partial}{partial y}Gamma(j,lambda y) =dfrac{partial}{partial (lambda y)}Gamma(j,lambda y) cdot dfrac{partial}{partial y}lambda y = -(lambda y)^{j-1}e^{-lambda y}lambda$$



So working the PDF a bit



$$begin{align*}p(y) &= q^Mdelta(y)+H(y) dfrac{p}{q} q^{M} lambda e^{-lambda y}sum_{j=0}^{M-1} binom{M}{j+1} left(dfrac{p}{q}lambda y right)^{j} dfrac{1}{j!}\
\
&= delta(y)q^M+H(y) lambda e^{-lambda y}sum_{j=0}^{M-1} binom{M}{j+1} q^{M-(j+1)}p^{j+1}left(lambda y right)^{j} dfrac{1}{j!}\
\
&= delta(y)q^M+H(y) lambda e^{-lambda y}sum_{j=1}^{M} binom{M}{j} q^{M-j}p^{j}dfrac{1}{(j-1)!}left(lambda y right)^{j-1} \
\
&= delta(y)q^M-H(y) sum_{j=1}^{M} binom{M}{j} q^{M-j}p^{j}dfrac{1}{(j-1)!}(-1)left(lambda y right)^{j-1} e^{-lambda y}lambda \
\
&= delta(y)q^M-H(y) sum_{j=1}^{M} binom{M}{j} q^{M-j}p^{j}dfrac{partial}{partial y}dfrac{Gamma(j,lambda y)}{Gamma(j)} \
end{align*}$$



To integrate the PDF to get the CDF, I'll be using the following relations:



$$int_{-infty}^{x} H(t)f'(t) dt = H(x)int_0^x f'(t) dt = H(x)left[f(x) - f(0)right]$$



$$int_{-infty}^x delta(t) f(t)dt = H(x)f(0)$$



So, integrating the PDF to find the CDF:



$$begin{align*}P(y) &= int_{-infty}^y p(t) dt\
\
&= int_{-infty}^y delta(t)q^M dt - sum_{j=1}^{M} binom{M}{j} q^{M-j}p^{j}int_{-infty}^y H(t)dfrac{partial}{partial t}dfrac{Gamma(j,lambda t)}{Gamma(j)} dt\
\
&= H(y)q^M -sum_{j=1}^{M} binom{M}{j} q^{M-j}p^{j} H(y)left[dfrac{Gamma(j,lambda y)}{Gamma(j)}-dfrac{Gamma(j,0)}{Gamma(j)}right]\
\
&= H(y)left[q^M -sum_{j=1}^{M} binom{M}{j} q^{M-j}p^{j} left(dfrac{Gamma(j,lambda y)}{Gamma(j)}-1right)right]\
\
&= H(y)left[q^M +sum_{j=1}^{M} binom{M}{j} q^{M-j}p^{j} -sum_{j=1}^{M} binom{M}{j} q^{M-j}p^{j}dfrac{Gamma(j,lambda y)}{Gamma(j)}right]\
\
&= H(y)left[(q+p)^M -sum_{j=1}^{M} binom{M}{j} q^{M-j}p^{j}dfrac{Gamma(j,lambda y)}{Gamma(j)}right]\
\
P(y) &= H(y)left[1 -sum_{j=1}^{M} binom{M}{j} q^{M-j}p^{j}dfrac{Gamma(j,lambda y)}{Gamma(j)}right]\
end{align*}$$






share|cite|improve this answer











$endgroup$



I was able to compute the integral you presented using contour integration in the complex plane.



For the case of $lambda > 0$, $y <0$, it is easy to show via contour integration that



$$int_{-infty}^infty dfrac{dk}{2pi}e^{-iyk}left(dfrac{ilambda p}{k - (-ilambda)}+qright)^M = 0$$



For the more interesting case of $lambda > 0$, $y > 0$, one can show via contour integration, and algebraic manipulations to generate a Laurent Series, that



$$begin{align*}int_{-infty}^infty dfrac{dk}{2pi}e^{-iyk}left(dfrac{ilambda p}{k - (-ilambda)}+qright)^M &= -2pi i space mathrm{Res}_{z=-ilambda}left[dfrac{1}{2pi}e^{-iyz}left(dfrac{ilambda p}{z - (-ilambda)}+qright)^Mright]\
\
&= -2pi i space mathrm{Res}_{z=-ilambda}left[dfrac{1}{2pi}e^{-iyz}sum_{j=0}^{M}binom{M}{j}left(dfrac{ilambda p}{z - (-ilambda)}right)^jq^{M-j}right]\
\
&= -2pi i space mathrm{Res}_{z=-ilambda}left[dfrac{1}{2pi}left(sum_{n=0}^infty dfrac{left(-iyright)^n e^{-lambda y}}{n!}left(z-left(-ilambdaright)right)^nright)left(sum_{j=0}^{M}binom{M}{j}left(dfrac{ilambda p}{z - (-ilambda)}right)^jq^{M-j}right)right]\
\
&= -2pi i dfrac{1}{2pi}sum_{j=1}^M dfrac{left(-iyright)^{j-1}e^{-lambda y}}{left(j-1right)!}binom{M}{j}left(ilambda pright)^jq^{M-j} \
\
&= sum_{j=1}^M binom{M}{j} p^j q^{M-j}dfrac{lambda^j y^{j-1} e^{-lambda y}}{left(j-1right)!}\
\
&mbox{(confirmation of exactly what you already thought to be true)}\
\
&= dfrac{q^M e^{-lambda y}}{y}sum_{j=1}^M binom{M}{j} left(dfrac{lambda y p}{q}right)^j dfrac{1}{left(j-1right)!}\
end{align*}$$



For the case of $lambda > 0$, $y =0$, the integral diverges due to the "$+q$". The integral diverges at this single value of $y$ with a weight of $q^M$, given some hindsight with the convolution theorem and the definition of a CDF.



The integral really does not provide any new information regarding the series. You need to manipulate the series form into something you find palatable.



The above development of the Laurent Series, to find the residue, does provide some insights into how portions of each term in the final sum arise. The binomial coefficients come from the binomial expansion, as you would expect. The factorial factors come from the Taylor Series expansion of the exponential function. The final summation comes from multiplying series terms together and adding to find the coefficient of the $(z-(-ilambda))^{-1}$ term of the Laurent Series.



Consolidating the results for all real values of $y$, the expression should really be written as:



$$begin{align*}p(y) &= q^Mdelta(y)+H(y)dfrac{q^M e^{-lambda y}}{y}sum_{j=1}^M binom{M}{j} left(dfrac{lambda y p}{q}right)^j dfrac{1}{left(j-1right)!}\
\
&= q^Mdelta(y)+ H(y) dfrac{p}{q} q^{M} lambda e^{-lambda y}sum_{j=0}^{M-1} binom{M}{j+1} left(dfrac{p}{q}lambda yright)^{j} dfrac{1}{j!}\
end{align*}$$



Where $H(y)$ is the Heaviside unit step and $delta(y)$ is the Dirac Delta function.



Using this expression, convolving the $M=1$ case with itself $M-1$ times, yields the correct expression for the $M^{th}$ case, as expected given the convolution theorem of the Fourier Transform.



Using the above expression for $p(y)$, we find



$$int_{-infty}^infty p(y) dy = (q+p)^M =1$$



which is what one would expect.



To replace the series, Wolfram Alpha comes up with this "closed" form in terms of a hypergeometric function:



$$begin{align*}p(y) &= q^Mdelta(y)+H(y) dfrac{p}{q} q^{M} lambda e^{-lambda y}sum_{j=0}^{M-1} binom{M}{j+1} left(dfrac{p}{q}lambda y right)^{j} dfrac{1}{j!}\
\
&= q^Mdelta(y)+ H(y) dfrac{p}{q} q^{M} lambda e^{-lambda y} M space {}_1F_1 left(1-M;2;-dfrac{p}{q}lambda yright)
end{align*}$$



Update to derive the CDF from the PDF



To derive the CDF, let's rewrite the PDF a little bit. I'll be using the following expression involving the Incomplete Upper Gamma Function in the process



$$dfrac{partial}{partial y}Gamma(j,lambda y) =dfrac{partial}{partial (lambda y)}Gamma(j,lambda y) cdot dfrac{partial}{partial y}lambda y = -(lambda y)^{j-1}e^{-lambda y}lambda$$



So working the PDF a bit



$$begin{align*}p(y) &= q^Mdelta(y)+H(y) dfrac{p}{q} q^{M} lambda e^{-lambda y}sum_{j=0}^{M-1} binom{M}{j+1} left(dfrac{p}{q}lambda y right)^{j} dfrac{1}{j!}\
\
&= delta(y)q^M+H(y) lambda e^{-lambda y}sum_{j=0}^{M-1} binom{M}{j+1} q^{M-(j+1)}p^{j+1}left(lambda y right)^{j} dfrac{1}{j!}\
\
&= delta(y)q^M+H(y) lambda e^{-lambda y}sum_{j=1}^{M} binom{M}{j} q^{M-j}p^{j}dfrac{1}{(j-1)!}left(lambda y right)^{j-1} \
\
&= delta(y)q^M-H(y) sum_{j=1}^{M} binom{M}{j} q^{M-j}p^{j}dfrac{1}{(j-1)!}(-1)left(lambda y right)^{j-1} e^{-lambda y}lambda \
\
&= delta(y)q^M-H(y) sum_{j=1}^{M} binom{M}{j} q^{M-j}p^{j}dfrac{partial}{partial y}dfrac{Gamma(j,lambda y)}{Gamma(j)} \
end{align*}$$



To integrate the PDF to get the CDF, I'll be using the following relations:



$$int_{-infty}^{x} H(t)f'(t) dt = H(x)int_0^x f'(t) dt = H(x)left[f(x) - f(0)right]$$



$$int_{-infty}^x delta(t) f(t)dt = H(x)f(0)$$



So, integrating the PDF to find the CDF:



$$begin{align*}P(y) &= int_{-infty}^y p(t) dt\
\
&= int_{-infty}^y delta(t)q^M dt - sum_{j=1}^{M} binom{M}{j} q^{M-j}p^{j}int_{-infty}^y H(t)dfrac{partial}{partial t}dfrac{Gamma(j,lambda t)}{Gamma(j)} dt\
\
&= H(y)q^M -sum_{j=1}^{M} binom{M}{j} q^{M-j}p^{j} H(y)left[dfrac{Gamma(j,lambda y)}{Gamma(j)}-dfrac{Gamma(j,0)}{Gamma(j)}right]\
\
&= H(y)left[q^M -sum_{j=1}^{M} binom{M}{j} q^{M-j}p^{j} left(dfrac{Gamma(j,lambda y)}{Gamma(j)}-1right)right]\
\
&= H(y)left[q^M +sum_{j=1}^{M} binom{M}{j} q^{M-j}p^{j} -sum_{j=1}^{M} binom{M}{j} q^{M-j}p^{j}dfrac{Gamma(j,lambda y)}{Gamma(j)}right]\
\
&= H(y)left[(q+p)^M -sum_{j=1}^{M} binom{M}{j} q^{M-j}p^{j}dfrac{Gamma(j,lambda y)}{Gamma(j)}right]\
\
P(y) &= H(y)left[1 -sum_{j=1}^{M} binom{M}{j} q^{M-j}p^{j}dfrac{Gamma(j,lambda y)}{Gamma(j)}right]\
end{align*}$$







share|cite|improve this answer














share|cite|improve this answer



share|cite|improve this answer








edited Dec 23 '18 at 17:54

























answered Dec 19 '18 at 1:16









Andy WallsAndy Walls

1,754139




1,754139












  • $begingroup$
    Solid @Andy. I especially appreciate the contour integration to show my two expressions are equivalent. That's satisfying. I guess the hypergeometric solution is as good as it gets. I was hoping for something simpler for the application which brought this problem up, but it is what it is.
    $endgroup$
    – kevinkayaks
    Dec 22 '18 at 2:36










  • $begingroup$
    Thanks. I just added an update to rework the PDF a little, so I could also derive the CDF.
    $endgroup$
    – Andy Walls
    Dec 23 '18 at 17:55










  • $begingroup$
    Yeah, there seems to be no easy way to simplify those $Gamma(,)$ related weights from the otherwise binomial expansion.
    $endgroup$
    – Andy Walls
    Dec 23 '18 at 18:01


















  • $begingroup$
    Solid @Andy. I especially appreciate the contour integration to show my two expressions are equivalent. That's satisfying. I guess the hypergeometric solution is as good as it gets. I was hoping for something simpler for the application which brought this problem up, but it is what it is.
    $endgroup$
    – kevinkayaks
    Dec 22 '18 at 2:36










  • $begingroup$
    Thanks. I just added an update to rework the PDF a little, so I could also derive the CDF.
    $endgroup$
    – Andy Walls
    Dec 23 '18 at 17:55










  • $begingroup$
    Yeah, there seems to be no easy way to simplify those $Gamma(,)$ related weights from the otherwise binomial expansion.
    $endgroup$
    – Andy Walls
    Dec 23 '18 at 18:01
















$begingroup$
Solid @Andy. I especially appreciate the contour integration to show my two expressions are equivalent. That's satisfying. I guess the hypergeometric solution is as good as it gets. I was hoping for something simpler for the application which brought this problem up, but it is what it is.
$endgroup$
– kevinkayaks
Dec 22 '18 at 2:36




$begingroup$
Solid @Andy. I especially appreciate the contour integration to show my two expressions are equivalent. That's satisfying. I guess the hypergeometric solution is as good as it gets. I was hoping for something simpler for the application which brought this problem up, but it is what it is.
$endgroup$
– kevinkayaks
Dec 22 '18 at 2:36












$begingroup$
Thanks. I just added an update to rework the PDF a little, so I could also derive the CDF.
$endgroup$
– Andy Walls
Dec 23 '18 at 17:55




$begingroup$
Thanks. I just added an update to rework the PDF a little, so I could also derive the CDF.
$endgroup$
– Andy Walls
Dec 23 '18 at 17:55












$begingroup$
Yeah, there seems to be no easy way to simplify those $Gamma(,)$ related weights from the otherwise binomial expansion.
$endgroup$
– Andy Walls
Dec 23 '18 at 18:01




$begingroup$
Yeah, there seems to be no easy way to simplify those $Gamma(,)$ related weights from the otherwise binomial expansion.
$endgroup$
– Andy Walls
Dec 23 '18 at 18:01


















draft saved

draft discarded




















































Thanks for contributing an answer to Mathematics Stack Exchange!


  • Please be sure to answer the question. Provide details and share your research!

But avoid



  • Asking for help, clarification, or responding to other answers.

  • Making statements based on opinion; back them up with references or personal experience.


Use MathJax to format equations. MathJax reference.


To learn more, see our tips on writing great answers.




draft saved


draft discarded














StackExchange.ready(
function () {
StackExchange.openid.initPostLogin('.new-post-login', 'https%3a%2f%2fmath.stackexchange.com%2fquestions%2f3039945%2fcompound-binomial-exponential-closed-form-for-the-pdf%23new-answer', 'question_page');
}
);

Post as a guest















Required, but never shown





















































Required, but never shown














Required, but never shown












Required, but never shown







Required, but never shown

































Required, but never shown














Required, but never shown












Required, but never shown







Required, but never shown







Popular posts from this blog

Plaza Victoria

In PowerPoint, is there a keyboard shortcut for bulleted / numbered list?

How to put 3 figures in Latex with 2 figures side by side and 1 below these side by side images but in...